ChaseDream
搜索
12
返回列表 发新帖
楼主: cranberry
打印 上一主题 下一主题

lsat-12-2-19

[复制链接]
11#
发表于 2004-8-25 10:52:00 | 只看该作者

我是在分析选项的倾向性,分析选项从那个角度去削弱原文的推理,并不是从原文推理出信息。对于A,完全有可能是其他原因(不是看电视的原因)使常看电视新闻的人比其他人更可能常看报纸,但是和是看电视的原因使常看电视新闻的人比其他人更可能常看报纸的可能性相比,绝不会是各50%,后者可能性要大得多,这就是A要表达的意思。就是这种较大的可能性去削弱原文的推理。正像MINDFREE和CRANBERRY所说的一样,单纯人的重叠不会削弱原文的ARGUMENT,因为讨论的问题不同,原文的讨论的是常看电视的后果。单纯的重叠只会削弱这样的结论:常看电视新闻的人比常看报纸的人看问题简单。这个结论不同于:相对于常看报纸,常看电视新闻使人看问题简单。前者比较两种人,后者比较两个动作的效果。要削弱的方向也不同,前者可以是有些人就是两者都有(重叠),所以无法比较,从而WEAKEN。后者要从动作的效果方面去削弱,说常看电视也不一定使人看问题简单。

12#
发表于 2004-8-25 11:07:00 | 只看该作者

单纯人的重叠不会削弱原文的ARGUMENT,因为讨论的问题不同,原文的讨论的是常看电视的后果。单纯的重叠只会削弱这样的结论:常看电视新闻的人比常看报纸的人看问题简单。

这段话不太明白;既然看电视新闻的人又看了报纸,那么可能性就有三种:1.是受看电视新闻的影响,看问题简单化;2.受两者影响,结果介于两者之间;3.受看报纸的影响。在这些可能存在的情况下,原文的结论还能一定成立吗?

13#
 楼主| 发表于 2004-8-25 17:21:00 | 只看该作者
我觉得这个答案只能是无奈之选,我们从答案中也无法明确看报纸到底是看电视的必然后果。如果可以得出看电视必然导致爱看报纸,则结论被削弱。毕竟其它的选项不沾边。
14#
发表于 2004-8-25 19:47:00 | 只看该作者
"原文的结论还能一定成立吗?"。“如果可以得出看电视必然导致爱看报纸,则结论被削弱”。这是WEAKEN题,不是ASSUMPTION题,只要可能性发生变化就可以,即使是一点点的变化。况且ASSUMPTION也不一定都MUST BE,只有充分类,必要类不需要,而多数是必要类,这就是为何要在问题中加MOST的原因,我见过不少人说MOST没意义,非也。
[此贴子已经被作者于2004-8-25 19:48:26编辑过]
15#
 楼主| 发表于 2004-8-25 20:02:00 | 只看该作者
是否只要提供的信息暗示一种可能性,比如一种和结论不同(而非对抗)的可能性就可以认为是削弱呢?我一直也认为,削弱是使得结论成立的可能降低,并非是一定要完全否定结论,但是在这个题目中,其结论也是增加一种可能和倾向,它并未排除相反的状况,所以很难说,结论和选项存在不同或冲突,因此也并不能降低结论成立的可能性。
16#
发表于 2004-8-25 20:31:00 | 只看该作者
17#
发表于 2019-7-31 18:43:06 | 只看该作者
cranberry 发表于 2004-8-24 22:35
The format of network television news programs generally allows advocates of a point of view only 30 ...

Spot the question type: Weaken

Within A, because of C happens, so no D.

Within B, because of E happens, so  D.

So as comparing group of A and B, it must be true that No D will always happen within group of A.

Inferences:

1. What if all People in group A happens to be in group B ?

2. What if people in group A would be also influenced by E ?

3. What if people in group A would find a way to avoid C happens ?

Answers:

A. people in Group A are likely to be the people in Group B => Matching our inferences

B. It does not impact the argument.

C. It supports the argument but not weaken it.

D. It does not impact the argument.

E. Support the argument.

您需要登录后才可以回帖 登录 | 立即注册

Mark一下! 看一下! 顶楼主! 感谢分享! 快速回复:

所属分类: 法学院申请

近期活动

正在浏览此版块的会员 ()

手机版|ChaseDream|GMT+8, 2024-5-27 05:47
京公网安备11010202008513号 京ICP证101109号 京ICP备12012021号

ChaseDream 论坛

© 2003-2023 ChaseDream.com. All Rights Reserved.

返回顶部